La ricerca ha trovato 486 risultati

da Gottinger95
30 lug 2014, 18:32
Forum: Combinatoria
Argomento: Cartiamo!
Risposte: 3
Visite : 2671

Re: Cartiamo!

Mmm, rispondo al punto a). Coloriamo gli assi di rosso e tutte le altre carte di blu. I valori delle carte ce li possiamo pure scordare. Le configurazioni in cui una carta rossa è alla \(k\)-esima posizione sono \( \displaystyle \binom{40-k}{3}\), ossia i modi di disporre le rimanenti \(40-k\) carte...
da Gottinger95
30 lug 2014, 12:48
Forum: Combinatoria
Argomento: Strategia vincente gara
Risposte: 9
Visite : 5455

Re: Strategia vincente gara

@Troileto: vedi, quando sei costretto a studiare fisica, ti convinci che tuffarti nelle cose più sporche sia entusiasmante.
@Draco: confermo, e in realtà i conti non sono nemmeno così brutti!
da Gottinger95
23 lug 2014, 16:39
Forum: Combinatoria
Argomento: Strategia vincente gara
Risposte: 9
Visite : 5455

Re: Strategia vincente gara

Io ho ragionato piú o meno così: supponi che l'auto si fermi al pit stop l'\(i\)-esima volta dopo \(a_i\) giri. Diciamo si ferma quindi dopo \(a_1, ..., a_k\) giri; la soma fa \(n_g\) (nel nostro caso 120). Riesci q dire il tempo totale in funzione della sequenza e di \(t_b, b_g,t_p, n_g\), rispetti...
da Gottinger95
21 lug 2014, 20:25
Forum: Teoria dei Numeri
Argomento: $x^2+y^2+z^2=2xy+2yz+2zx$
Risposte: 7
Visite : 7508

Re: $x^2+y^2+z^2=2xy+2yz+2zx$

Si, scusa, mi sono dimenticato di aggiungere \(x \neq y\) (che discende dalle prime due righe).
da Gottinger95
21 lug 2014, 20:24
Forum: Teoria dei Numeri
Argomento: Cortona 95
Risposte: 15
Visite : 9399

Re: Cortona 95

Grande gpzes! Daje così :) Manca solo di dimostrare che con un \(m\) più piccolo non si può fare
da Gottinger95
21 lug 2014, 20:22
Forum: Teoria dei Numeri
Argomento: Quante somme di quadrati!
Risposte: 2
Visite : 4531

Re: Quante somme di quadrati!

( Atttenzione, spoiler in the testo nascosto !) Per quanto riguarda il bonus, in generale vale \(S(n) = D_1(n) - D_3(n)\), dove: - \(S(n)= |\{ (a,b): \ \ a,b \in \mathbb{N} \ \ a^2+b^2 = n\}|\); - \(D_1(n) = |\{ d \mid n: \ \ d \equiv 1 \pmod{4} \}| \), e \(D_3(n)\) definito similmente. Quindi la ri...
da Gottinger95
21 lug 2014, 14:17
Forum: Teoria dei Numeri
Argomento: La funzione di Eulero ha parecchi buchi
Risposte: 2
Visite : 5251

Re: La funzione di Eulero ha parecchi buchi

Visto che nessuno posta, intanto scrivo io una soluzione puzzona e un po' barona. Scelto \(r\) tale che \(2^{r+1} > k\), imponiamo che \( n \not \equiv -1, -2, \ldots, -k \pmod{2^{r+1} } \ \ (*) \). In questo modo, visto che \( \omega(m)=s \ \ \Rightarrow \ \ 2^s \mid \varphi(m) \), sicuramente la s...
da Gottinger95
21 lug 2014, 02:26
Forum: Teoria dei Numeri
Argomento: Cortona 95
Risposte: 15
Visite : 9399

Re: Cortona 95

@xXStephXx: tutto giusto, solo piccoli dettagli "formali": - avevo messo \(< m\), quindi va sostituito un \(m\) a tutti gli \(m+1\) (ma questa è solo estetica, diciamocelo); - avevo fissato \(n,k\), perciò la condizione era su \(m\), perciò il risultato è \( m \ge \sqrt[k]{n} \); nella pri...
da Gottinger95
20 lug 2014, 15:24
Forum: Teoria dei Numeri
Argomento: Cortona 95
Risposte: 15
Visite : 9399

Re: Cortona 95

@gzpes: \(1,2,4, \ldots, 512\): nessuna assegnazione di \(0,\pm 1\) produce un numero divisibile per \(1024\). \(1,1023,4, \ldots, 512\): assegnando \(+1, +1, 0, \ldots, 0 \) si produce un numero divisibile per \(1024\). Che intendi con il tuo "ma anche"? Rilancio: Siano \(k,n \in \mathbb{...
da Gottinger95
17 lug 2014, 15:21
Forum: Teoria dei Numeri
Argomento: Cortona 95
Risposte: 15
Visite : 9399

Re: Cortona 95

Il problema si può riformulare con \(k\) al posto di \(10\) e \(n\) al posto di 1001 in questo modo. Siano \(n,k \in \mathbb{N} \) e sia \(A = \{1, \ldots, k\}\). Dati \(a_1, \ldots, a_k \in \mathbb{N} \), trovare (se esistono) \( S_1, S_2 \subseteq A\) tali che \[ \sum_{i \in S_1} a_i = \sum_{i \in...
da Gottinger95
15 lug 2014, 18:12
Forum: Algebra
Argomento: Stimare somme parziali
Risposte: 4
Visite : 3051

Stimare somme parziali

Sia \(f(x)\) una funzione concava e crescente in \([0,N]\). Dimostrare che \[ \int_0^N f(x) dx \le \sum_{k=1}^N f(k) \le \int_0^N f(x) dx + \frac{f(N) - f(0)}{2} \] N.B. Sugli integrali non serve sapere praticamente nulla, se non queste proprietà base (scrivo per chi non li conosce): 1. L'integrale ...
da Gottinger95
15 lug 2014, 15:21
Forum: Matematica non elementare
Argomento: so qualcosa sulla derivata...
Risposte: 38
Visite : 25108

Re: so qualcosa sulla derivata...

E chi ti dice che \(S_f\) abbia massimo? Eh, infatti la mia domanda era proprio se "la derivata da un certo punto in poi si annulla in tutti i punti" significasse che \(S_f\) fosse limitato. A quanto pare no! :D In realtà dimostrare che \(S_f\) abbia massimo è equivalente a dimostrare che...
da Gottinger95
14 lug 2014, 17:40
Forum: Matematica non elementare
Argomento: so qualcosa sulla derivata...
Risposte: 38
Visite : 25108

Re: so qualcosa sulla derivata...

Domanda da ignorantone: sia \[ S_f = \{ k_x: \ x \in \mathbb{R}, \ |f^{(k_x-1)}(x)| > 0, \ \forall k \ge k_x \ \ f^{(k)}(x) = 0 \} = \{\mbox{insieme dei punti dove si iniziano ad annullare le derivate} \} \] Per come è posto il problema, si intende che \(S_f\) sia limitato? Non credo eh, però non si...
da Gottinger95
10 lug 2014, 02:06
Forum: Algebra
Argomento: Ooh che monotonia queste funzioni
Risposte: 6
Visite : 3418

Re: Ooh che monotonia queste funzioni

E' vero, quanta saggezza, sono stato un po' frettoloso! Modulo conti, ho capito il concetto. Però che bello, sono ben più contento di questa risposta che di quella che mi aspettavo. Grazie a entrambi! Per curiosità: è vero anche il contrario? Cioè se invece \(f'(x) \) è limitata allora \(f(x)\) è BV...
da Gottinger95
09 lug 2014, 20:07
Forum: Algebra
Argomento: Ooh che monotonia queste funzioni
Risposte: 6
Visite : 3418

Re: Ooh che monotonia queste funzioni

Mmm. Mmm. Non capisco perchè \[\Delta f\left[\frac{1}{2k+1},\frac{1}{2k}\right] = \frac{1}{2k+1}+\frac{1}{2k} \] Non mi pare che corrisponda alle definizioni di \(f, \Delta f\). Comunque sono certo che sono ottuso, e la cosa che hai scritto sarà fuor di dubbio ragionevole. Ma allora non mi quadra qu...